Q12

 
unclefester2013
Thanks Received: 1
Vinny Gambini
Vinny Gambini
 
Posts: 16
Joined: July 01st, 2009
 
 
trophy
First Responder
 

Q12

by unclefester2013 Mon Nov 30, 2009 10:41 pm

I am confused by the wording of Question #12 on this game as well. If anyone could shed some light, it would be appreciated.
 
mhallett06
Thanks Received: 0
Forum Guests
 
Posts: 2
Joined: May 16th, 2010
 
 
 

Re: PT 58, S3, G2 - A company Organizing on site day care

by mhallett06 Sun Jun 06, 2010 12:50 pm

Hello,

I am still having trouble with 12. I correctly guessed B but when I went back to review the game I incorrectly guessed C.

After performing all of my inferences and several possible groups I came up with:

In Out
FL S/V T, M, R
RMTLS FV
LVFR STM

How do I eliminate C?

Thank you in advance.

One more day :-)
User avatar
 
ManhattanPrepLSAT1
Thanks Received: 1909
Atticus Finch
Atticus Finch
 
Posts: 2851
Joined: October 07th, 2009
 
This post thanked 1 time.
 
 

Re: PT 58, S3, G2 - A company Organizing on site day care

by ManhattanPrepLSAT1 Mon Jun 07, 2010 1:15 am

Here's a hypothetical that satisfies all of the constraints and yet has neither L nor V selected. This should help get rid of answer choice (C).

IN: R M T S
OUT: L V F

Another way to see this is that rules that move from negative to positive, such as the 3rd and 4th constraints imply that at least one of the pair must be selected.

So from the 3rd constraint it must be true that at least one of S and V are selected.

From the 4th constraint it must be true that at least one of R and L are selected.

Unfortunately, neither of those pairs are answer choices. But if you combine the the 1st and the 4th constraints the following inference can be made

~M --> L

This would imply that at least one of M and L must be selected which is answer choice (B)! I hope this helps and good luck tomorrow. Don't let other peoples' stress get to you. Stay focused on the prize...
 
joyce.k.hahn
Thanks Received: 0
Forum Guests
 
Posts: 1
Joined: November 28th, 2010
 
 
 

Re: PT 58, S3, G2 - A company Organizing on site day care

by joyce.k.hahn Tue Dec 07, 2010 8:29 pm

I narrowed down my choices to A and B and only guessed A because I didn't want to leave a blank. Can you explain why A is incorrect if one of the conditions states "If Terry volunteers, then neither Felicia nor Veena volunteers"? Isn't this relationship expressed as T --> -F and the contrapositive F --> -T, meaning at least one volunteers?
 
tom
Thanks Received: 0
Forum Guests
 
Posts: 1
Joined: December 15th, 2010
 
 
 

Re: PT 58, S3, G2 - A company Organizing on site day care

by tom Wed Dec 15, 2010 1:16 pm

joyce.k.hahn Wrote:I narrowed down my choices to A and B and only guessed A because I didn't want to leave a blank. Can you explain why A is incorrect if one of the conditions states "If Terry volunteers, then neither Felicia nor Veena volunteers"? Isn't this relationship expressed as T --> -F and the contrapositive F --> -T, meaning at least one volunteers?


You've basically answered your own question. The question is asking, no matter what, who are the two where one will still volunteer?

F>V>-T>-M>-R> L
R>M>T>-F>-V>S> L (rules don't affect L)

That was my basic diagram. A is wrong because at least one of them isn't staying as a volunteer (T and F fluctuate between volunteering and not volunteering). L, on the other hand, stays rock solid and is always volunteering. That narrows your choices down to B&C. And if you want to cheat a bit, L is a quasi-wildcard, so it's a dead giveaway that the answer will contain L.

You can eliminate answer C because as the post above indicates, you can stick L and V into the not volunteer group and not violate the rules. That's no good because you'll have a circumstance where both are, or not volunteers when exposed to different variables. You're looking for at least one volunteer (L) that will remain CONSTANT regardless.

You have more flexibility with answer B to keep L as the volunteer regardless of what happens to M.

This is a tough question. I've spent the last few hours wrapping my head around this one question. The rest in this group were fairly straight forward. LSAT is in February...oy!
User avatar
 
ManhattanPrepLSAT1
Thanks Received: 1909
Atticus Finch
Atticus Finch
 
Posts: 2851
Joined: October 07th, 2009
 
This post thanked 1 time.
 
 

Re: PT 58, S3, G2 - A company Organizing on site day care

by ManhattanPrepLSAT1 Mon Dec 20, 2010 4:39 am

joyce.k.hahn Wrote:Can you explain why A is incorrect if one of the conditions states "If Terry volunteers, then neither Felicia nor Veena volunteers"? Isn't this relationship expressed as T --> -F and the contrapositive F --> -T, meaning at least one volunteers?

Be careful about the way you're interpreting the constraint. It does not imply that at least one volunteers but rather that at most one volunteers. Rules that mover from positive to negative imply that at most one of the two can occur, whereas rules that move from negative to positive imply that at least one occurs.

So for example (A ---> ~B) would imply that one of A or B could occur or that neither could occur. What's not possible is that both occur. And (~A ---> B) would imply that one or the other of A and B could occur or both could occur. What's not possible in this case is that neither would occur.

To answer this question you're looking for a constraint that moves from negative to positive between to two characters. The third and fourth constraints do this, but neither of those pairs are answer choices.

Using the contrapositive of the fourth constraint and the first constraint:

~L ---> R
R ---> M

We can infer

~L ---> M

This guarantees that at least one of L and M will be in, which is answer choice (B).

Does that answer your question?

This impl
 
chike_eze
Thanks Received: 94
Atticus Finch
Atticus Finch
 
Posts: 279
Joined: January 22nd, 2011
 
This post thanked 3 times.
 
trophy
Most Thanked
 

Re: Q12

by chike_eze Wed Nov 09, 2011 3:34 am

To be honest, I didn't know where to start with this question -- cos I had no idea what it was referring to.

But after reviewing for 2 days now, I finally recall that I've seen this question before, albeit phrased differently.

It is basically asking, which one of these pairs cannot both not volunteer? i.e., which pair cannot both be out?

If so, then the most efficient path to the correct answer is to attempt to force each pair out. The pair that cannot both be out is the correct answer.

E.g.

A) out: F, T
> in:L out:R,M
> yes no rule violated here

B) out: L, M
> in:R,M,T out:F,V,S,M
O wait, per rule combination, if L is out -> M must be in (and vice-versa)
Stop! that's the answer -- L and M cannot both be out.

No need to test the rest.

For laughs:

C) out: L, V
> in: R,M,T,S out: F,V
> no rule violated here

D) out: R, S
> in:L,V out:R,M,T
> no rule violated here

E) out: S, T
> in:V,L out:R,M,T
> no rule violated here